Proving that a sequence of reciprocated integers is not Cauchy [on hold]Proving a Sequence is CauchyProving...

Called into a meeting and told we are being made redundant (laid off) and "not to share outside". Can I tell my partner?

When to use the term transposed instead of modulation?

The past tense for the quoting particle って

Computing the volume of a simplex-like object with constraints

How to make sure I'm assertive enough in contact with subordinates?

Giving a talk in my old university, how prominently should I tell students my salary?

3.5% Interest Student Loan or use all of my savings on Tuition?

Why can't we use freedom of speech and expression to incite people to rebel against government in India?

How spaceships determine each other's mass in space?

Naming Characters after Friends/Family

Create chunks from an array

Do natural melee weapons (from racial traits) trigger Improved Divine Smite?

What is the purpose of a disclaimer like "this is not legal advice"?

Error in TransformedField

Is this nominative case or accusative case?

Are Wave equations equivalent to Maxwell equations in free space?

How to write a chaotic neutral protagonist and prevent my readers from thinking they are evil?

Why are special aircraft used for the carriers in the United States Navy?

Why is there an extra space when I type "ls" on the Desktop?

“I had a flat in the centre of town, but I didn’t like living there, so …”

Can inspiration allow the Rogue to make a Sneak Attack?

Is divide-by-zero a security vulnerability?

Quitting employee has privileged access to critical information

If nine coins are tossed, what is the probability that the number of heads is even?



Proving that a sequence of reciprocated integers is not Cauchy [on hold]


Proving a Sequence is CauchyProving that $(x_n)_{n=1}^{infty}$ is a Cauchy sequence.Prove the following sequence is a Cauchy Sequencecauchy sequence integersCauchy Sequences--is the floor function of a Cauchy sequence also a Cauchy sequence?sequence is cauchyProving a Sequence Converges - Cauchy?Cauchy sequence!Show that a sequence is not a Cauchy SequenceProving that a sequence is not Cauchy













-1












$begingroup$


Let ${a_n}_n$ be the sequence given by $a_n = 1 + frac 12 + cdots + frac 1n.$ Prove the sequence is not Cauchy.



I don't really have any idea where to start with this problem. I know the sequence diverges, but I can't think of any way to prove this one. Appreciate any help, thank you.










share|cite|improve this question









$endgroup$



put on hold as off-topic by RRL, YiFan, Leucippus, Eevee Trainer, Xander Henderson 45 mins ago


This question appears to be off-topic. The users who voted to close gave this specific reason:


  • "This question is missing context or other details: Please provide additional context, which ideally explains why the question is relevant to you and our community. Some forms of context include: background and motivation, relevant definitions, source, possible strategies, your current progress, why the question is interesting or important, etc." – RRL, YiFan, Leucippus, Eevee Trainer, Xander Henderson

If this question can be reworded to fit the rules in the help center, please edit the question.





















    -1












    $begingroup$


    Let ${a_n}_n$ be the sequence given by $a_n = 1 + frac 12 + cdots + frac 1n.$ Prove the sequence is not Cauchy.



    I don't really have any idea where to start with this problem. I know the sequence diverges, but I can't think of any way to prove this one. Appreciate any help, thank you.










    share|cite|improve this question









    $endgroup$



    put on hold as off-topic by RRL, YiFan, Leucippus, Eevee Trainer, Xander Henderson 45 mins ago


    This question appears to be off-topic. The users who voted to close gave this specific reason:


    • "This question is missing context or other details: Please provide additional context, which ideally explains why the question is relevant to you and our community. Some forms of context include: background and motivation, relevant definitions, source, possible strategies, your current progress, why the question is interesting or important, etc." – RRL, YiFan, Leucippus, Eevee Trainer, Xander Henderson

    If this question can be reworded to fit the rules in the help center, please edit the question.



















      -1












      -1








      -1





      $begingroup$


      Let ${a_n}_n$ be the sequence given by $a_n = 1 + frac 12 + cdots + frac 1n.$ Prove the sequence is not Cauchy.



      I don't really have any idea where to start with this problem. I know the sequence diverges, but I can't think of any way to prove this one. Appreciate any help, thank you.










      share|cite|improve this question









      $endgroup$




      Let ${a_n}_n$ be the sequence given by $a_n = 1 + frac 12 + cdots + frac 1n.$ Prove the sequence is not Cauchy.



      I don't really have any idea where to start with this problem. I know the sequence diverges, but I can't think of any way to prove this one. Appreciate any help, thank you.







      real-analysis cauchy-sequences






      share|cite|improve this question













      share|cite|improve this question











      share|cite|improve this question




      share|cite|improve this question










      asked 20 hours ago









      B RetnikB Retnik

      534




      534




      put on hold as off-topic by RRL, YiFan, Leucippus, Eevee Trainer, Xander Henderson 45 mins ago


      This question appears to be off-topic. The users who voted to close gave this specific reason:


      • "This question is missing context or other details: Please provide additional context, which ideally explains why the question is relevant to you and our community. Some forms of context include: background and motivation, relevant definitions, source, possible strategies, your current progress, why the question is interesting or important, etc." – RRL, YiFan, Leucippus, Eevee Trainer, Xander Henderson

      If this question can be reworded to fit the rules in the help center, please edit the question.







      put on hold as off-topic by RRL, YiFan, Leucippus, Eevee Trainer, Xander Henderson 45 mins ago


      This question appears to be off-topic. The users who voted to close gave this specific reason:


      • "This question is missing context or other details: Please provide additional context, which ideally explains why the question is relevant to you and our community. Some forms of context include: background and motivation, relevant definitions, source, possible strategies, your current progress, why the question is interesting or important, etc." – RRL, YiFan, Leucippus, Eevee Trainer, Xander Henderson

      If this question can be reworded to fit the rules in the help center, please edit the question.






















          2 Answers
          2






          active

          oldest

          votes


















          3












          $begingroup$

          We have $a_{2n}=a_n+frac{1}{n+1}+...+frac{1}{2n} ge a_n+n cdot frac{1}{2n}=a_n+frac{1}{2}.$



          Hence $|a_{2n}-a_n|=a_{2n}-a_n ge frac{1}{2}$ for all $n$.



          Can you take it from here ?






          share|cite|improve this answer









          $endgroup$





















            4












            $begingroup$

            $frac 1 n+frac 1 {n+1} +cdots+frac 1 {k_n} geq frac {k_n-n+1} {k_n}$ if $k_n >n$. We can choose $k_n$ such that $frac {k_n-n+1} {k_n}$ does not tend to $0$, e.g. $k_n=n^{2}$.






            share|cite|improve this answer









            $endgroup$













            • $begingroup$
              Why is $frac 1n + frac 1{n+1} + cdots + frac 1{k_n} ge frac {k_n-n+1}{k_n}$?
              $endgroup$
              – B Retnik
              19 hours ago






            • 2




              $begingroup$
              Because there are $k_n-n+1$ terms and each term is $geq frac 1 {k_n}$.
              $endgroup$
              – Kavi Rama Murthy
              19 hours ago


















            2 Answers
            2






            active

            oldest

            votes








            2 Answers
            2






            active

            oldest

            votes









            active

            oldest

            votes






            active

            oldest

            votes









            3












            $begingroup$

            We have $a_{2n}=a_n+frac{1}{n+1}+...+frac{1}{2n} ge a_n+n cdot frac{1}{2n}=a_n+frac{1}{2}.$



            Hence $|a_{2n}-a_n|=a_{2n}-a_n ge frac{1}{2}$ for all $n$.



            Can you take it from here ?






            share|cite|improve this answer









            $endgroup$


















              3












              $begingroup$

              We have $a_{2n}=a_n+frac{1}{n+1}+...+frac{1}{2n} ge a_n+n cdot frac{1}{2n}=a_n+frac{1}{2}.$



              Hence $|a_{2n}-a_n|=a_{2n}-a_n ge frac{1}{2}$ for all $n$.



              Can you take it from here ?






              share|cite|improve this answer









              $endgroup$
















                3












                3








                3





                $begingroup$

                We have $a_{2n}=a_n+frac{1}{n+1}+...+frac{1}{2n} ge a_n+n cdot frac{1}{2n}=a_n+frac{1}{2}.$



                Hence $|a_{2n}-a_n|=a_{2n}-a_n ge frac{1}{2}$ for all $n$.



                Can you take it from here ?






                share|cite|improve this answer









                $endgroup$



                We have $a_{2n}=a_n+frac{1}{n+1}+...+frac{1}{2n} ge a_n+n cdot frac{1}{2n}=a_n+frac{1}{2}.$



                Hence $|a_{2n}-a_n|=a_{2n}-a_n ge frac{1}{2}$ for all $n$.



                Can you take it from here ?







                share|cite|improve this answer












                share|cite|improve this answer



                share|cite|improve this answer










                answered 19 hours ago









                FredFred

                47.7k1849




                47.7k1849























                    4












                    $begingroup$

                    $frac 1 n+frac 1 {n+1} +cdots+frac 1 {k_n} geq frac {k_n-n+1} {k_n}$ if $k_n >n$. We can choose $k_n$ such that $frac {k_n-n+1} {k_n}$ does not tend to $0$, e.g. $k_n=n^{2}$.






                    share|cite|improve this answer









                    $endgroup$













                    • $begingroup$
                      Why is $frac 1n + frac 1{n+1} + cdots + frac 1{k_n} ge frac {k_n-n+1}{k_n}$?
                      $endgroup$
                      – B Retnik
                      19 hours ago






                    • 2




                      $begingroup$
                      Because there are $k_n-n+1$ terms and each term is $geq frac 1 {k_n}$.
                      $endgroup$
                      – Kavi Rama Murthy
                      19 hours ago
















                    4












                    $begingroup$

                    $frac 1 n+frac 1 {n+1} +cdots+frac 1 {k_n} geq frac {k_n-n+1} {k_n}$ if $k_n >n$. We can choose $k_n$ such that $frac {k_n-n+1} {k_n}$ does not tend to $0$, e.g. $k_n=n^{2}$.






                    share|cite|improve this answer









                    $endgroup$













                    • $begingroup$
                      Why is $frac 1n + frac 1{n+1} + cdots + frac 1{k_n} ge frac {k_n-n+1}{k_n}$?
                      $endgroup$
                      – B Retnik
                      19 hours ago






                    • 2




                      $begingroup$
                      Because there are $k_n-n+1$ terms and each term is $geq frac 1 {k_n}$.
                      $endgroup$
                      – Kavi Rama Murthy
                      19 hours ago














                    4












                    4








                    4





                    $begingroup$

                    $frac 1 n+frac 1 {n+1} +cdots+frac 1 {k_n} geq frac {k_n-n+1} {k_n}$ if $k_n >n$. We can choose $k_n$ such that $frac {k_n-n+1} {k_n}$ does not tend to $0$, e.g. $k_n=n^{2}$.






                    share|cite|improve this answer









                    $endgroup$



                    $frac 1 n+frac 1 {n+1} +cdots+frac 1 {k_n} geq frac {k_n-n+1} {k_n}$ if $k_n >n$. We can choose $k_n$ such that $frac {k_n-n+1} {k_n}$ does not tend to $0$, e.g. $k_n=n^{2}$.







                    share|cite|improve this answer












                    share|cite|improve this answer



                    share|cite|improve this answer










                    answered 20 hours ago









                    Kavi Rama MurthyKavi Rama Murthy

                    65.4k42766




                    65.4k42766












                    • $begingroup$
                      Why is $frac 1n + frac 1{n+1} + cdots + frac 1{k_n} ge frac {k_n-n+1}{k_n}$?
                      $endgroup$
                      – B Retnik
                      19 hours ago






                    • 2




                      $begingroup$
                      Because there are $k_n-n+1$ terms and each term is $geq frac 1 {k_n}$.
                      $endgroup$
                      – Kavi Rama Murthy
                      19 hours ago


















                    • $begingroup$
                      Why is $frac 1n + frac 1{n+1} + cdots + frac 1{k_n} ge frac {k_n-n+1}{k_n}$?
                      $endgroup$
                      – B Retnik
                      19 hours ago






                    • 2




                      $begingroup$
                      Because there are $k_n-n+1$ terms and each term is $geq frac 1 {k_n}$.
                      $endgroup$
                      – Kavi Rama Murthy
                      19 hours ago
















                    $begingroup$
                    Why is $frac 1n + frac 1{n+1} + cdots + frac 1{k_n} ge frac {k_n-n+1}{k_n}$?
                    $endgroup$
                    – B Retnik
                    19 hours ago




                    $begingroup$
                    Why is $frac 1n + frac 1{n+1} + cdots + frac 1{k_n} ge frac {k_n-n+1}{k_n}$?
                    $endgroup$
                    – B Retnik
                    19 hours ago




                    2




                    2




                    $begingroup$
                    Because there are $k_n-n+1$ terms and each term is $geq frac 1 {k_n}$.
                    $endgroup$
                    – Kavi Rama Murthy
                    19 hours ago




                    $begingroup$
                    Because there are $k_n-n+1$ terms and each term is $geq frac 1 {k_n}$.
                    $endgroup$
                    – Kavi Rama Murthy
                    19 hours ago



                    Popular posts from this blog

                    Magento 2 - Add success message with knockout Planned maintenance scheduled April 23, 2019 at 23:30 UTC (7:30pm US/Eastern) Announcing the arrival of Valued Associate #679: Cesar Manara Unicorn Meta Zoo #1: Why another podcast?Success / Error message on ajax request$.widget is not a function when loading a homepage after add custom jQuery on custom themeHow can bind jQuery to current document in Magento 2 When template load by ajaxRedirect page using plugin in Magento 2Magento 2 - Update quantity and totals of cart page without page reload?Magento 2: Quote data not loaded on knockout checkoutMagento 2 : I need to change add to cart success message after adding product into cart through pluginMagento 2.2.5 How to add additional products to cart from new checkout step?Magento 2 Add error/success message with knockoutCan't validate Post Code on checkout page

                    Fil:Tokke komm.svg

                    Where did Arya get these scars? Unicorn Meta Zoo #1: Why another podcast? Announcing the arrival of Valued Associate #679: Cesar Manara Favourite questions and answers from the 1st quarter of 2019Why did Arya refuse to end it?Has the pronunciation of Arya Stark's name changed?Has Arya forgiven people?Why did Arya Stark lose her vision?Why can Arya still use the faces?Has the Narrow Sea become narrower?Does Arya Stark know how to make poisons outside of the House of Black and White?Why did Nymeria leave Arya?Why did Arya not kill the Lannister soldiers she encountered in the Riverlands?What is the current canonical age of Sansa, Bran and Arya Stark?